LSAT and Law School Admissions Forum

Get expert LSAT preparation and law school admissions advice from PowerScore Test Preparation.

User avatar
 Dave Killoran
PowerScore Staff
  • PowerScore Staff
  • Posts: 5972
  • Joined: Mar 25, 2011
|
#80544
This game is also discussed in our Podcast: LSAT Podcast Episode 70: The May 2020 LSAT-Flex Logic Games Section

Complete Question Explanation
(The complete setup for this game can be found here: viewtopic.php?t=33052)

The correct answer choice is (A).

The question stem places L on day 7 and O on day 8. L on day 7 immediately triggers the second rule, placing L on day 2 as well:


  • ___ ..... _L_ ..... ___ ..... ___ ..... ___ ..... ___ ..... _L_ ..... _O_
     1 .....     2 .....    3 .....    4 .....     5 .....    6 .....      7 .....   8
This satisfies the second rule, as well as the third rule (since O is performed on day 8 and is walled off from M by L on day 7). However, the game is still quite open at this point. So, the best approach is to try a hypothetical or two.

To make hypotheticals here, don't just randomly start plugging in variables to spaces. Instead, focus on the areas of greatest effect. The first rule still needs requirement, and since it takes up two spaces (1-5) AND has Not Laws (H), it's a great starting point. Only three options exist for the 1-5 spaces: G, J, and M. Since G is a random, start with J and M first (side note: come back to this point later and consider why we bypassed G for now—as a random it is very weak; J and M are far more powerful, and thus better to start with for hypotheticals):


J is performed on days 1-5

With J in 1-5, it's immediately apparent that only days 3-4 are available for the HM block. That then forces the last variable, G, into day 6.


  • _J_ ..... _L_ ..... _H_ ..... _M_ ..... _J_ ..... _G_ ..... _L_ ..... _O_
     1 .....     2 .....    3 .....         4 .....     5 .....    6 .....      7 .....      8
Regrettably, this hypothetical is only good enough to eliminate answer choice (E). So, try another one!


M is performed on days 1-5

With M in 1-5, the only possible position of H to satisfy the fourth rule is on day 4. that leaves J and G to be placed. Because J must be performed before the first instance of H per the last rule, that forces J to be performed on day 3. G is then left to perform on day 6.

  • _M_ ..... _L_ ..... _J_ ..... _H_ ..... _M_ ..... _G_ ..... _L_ ..... _O_
     1 .....        2 .....    3 .....        4 .....     5 .....    6 .....      7 .....      8
This hypothetical has a greater effect than the first hypothetical, and it eliminates answer choices (B), (C), and (D).

Answer choice (A) is now the only remaining answer that has not been eliminated, and as further confirmation, both of our hypotheticals conform to answer choice (A)


Answer choice (A): This is the correct answer choice, and is proven correct by process of elimination as shown above.

Answer choice (B): While this answer choice could be true, it does not have to be true, and is thus incorrect.

Answer choice (C): While this answer choice could be true, it does not have to be true, and is thus incorrect.

Answer choice (D): While this answer choice could be true, it does not have to be true, and is thus incorrect.

Answer choice (E): While this answer choice could be true, it does not have to be true, and is thus incorrect.
 lsatryan
  • Posts: 8
  • Joined: Nov 09, 2020
|
#82631
Why is answer C incorrect?

Would the following work?

J L H M J H L O

Thank you
 Adam Tyson
PowerScore Staff
  • PowerScore Staff
  • Posts: 5387
  • Joined: Apr 14, 2011
|
#82750
It's important to consider the type of question here, lsatryan. This is a Must Be True question, so we shouldn't be asking ourselves "could this work?" We should be asking ourselves "is this required?" J going first is an option, but not a requirement in this local scenario, and that is why answer C is incorrect. Why isn't it required? Because there is another possibility where M goes first (MLJHMGLO).

Also, I think in your hypo you probably meant to put G in the 6th position, because as written you have no G in the answer and it must be included somewhere. Once you correct that mistake you will have further evidence that no matter what else happens when L is 7th and O is 8th, G must be 6th, which is answer A and is the correct answer.
 bilingli@gmail.com
  • Posts: 3
  • Joined: Nov 07, 2022
|
#100857
For the "To make hypotheticals here, don't just randomly start plugging in variables to spaces. Instead, focus on the areas of greatest effect. The first rule still needs requirement, and since it takes up two spaces (1-5) AND has Not Laws (H), it's a great starting point. Only three options exist for the 1-5 spaces: G, J, and M. Since G is a random, start with J and M first (side note: come back to this point later and consider why we bypassed G for now—as a random it is very weak; J and M are far more powerful, and thus better to start with for hypotheticals)" would a better approach be to concentrate on where the HM block can go? Less plugging this way.
User avatar
 Hanin Abu Amara
PowerScore Staff
  • PowerScore Staff
  • Posts: 60
  • Joined: Mar 29, 2023
|
#100862
That would work well! Anytime we say to start plugging and creating hypotheticals you do that in the way that works best for you. I prefer to start with the rule that has the most impact so I might start by splitting based on 1/5 like you are. Some people identify better rules that have a lot of impact and they use those.

As long as you're getting the right answer, where you start diagramming your hypotheticals isn't too critical.

Get the most out of your LSAT Prep Plus subscription.

Analyze and track your performance with our Testing and Analytics Package.